Are archimedean subextensions of ordered fields dense?












11














Let $E$ be an ordered field and let $F$ be a real closed subfield. We say that $E$ is $F$-archimedean if for each $ein E$ there is $xin F$ such that $-xle ele x$.




Is it true that if $E$ is $F$-archimedean then every interval in $E$ contains an element in $F$? That is, is it true that for every $e<e'$ in $E$ there is an element $xin F$ such that $e<x<e'$?






This is known if $F$ is the field of real algebraic numbers (in which case $E$ is an ordered subfield of $mathbb{R}$), and it seems to me that it should have an easy proof in the general case. However I cannot find neither an easy proof nor a counterexample.










share|cite|improve this question




















  • 2




    Have you tried $E = mathbb{Q}((infty)), F = mathbb{Q}((infty^2))$?
    – user44191
    Dec 17 at 19:49






  • 1




    What is $mathbb{Q}((infty))$? Do you mean $mathbb{Q}(t)$ with the order making $t$ an infinite positive element (I'm not sure this order can be extended to the Laurent series)?
    – Denis Nardin
    Dec 17 at 19:50










  • Yes, sorry, my idiocy.
    – user44191
    Dec 17 at 19:51










  • No need to be self-deprecating it's a good counterexample... Uhm there might in fact be no number between $t$ and $t+1$. Can you do a counterexample where $F$ is real closed? Sorry for changing the goal posts, I should have put it from the beginning since I was really thinking about that case.
    – Denis Nardin
    Dec 17 at 19:52








  • 1




    Try $E$ the real closure of $mathbb{Q}(x, y)$ with $x > mathbb{Q}, y > mathbb{Q}(x)$, and $F$ the real closure of $mathbb{Q}(y)$?
    – user44191
    Dec 17 at 20:00


















11














Let $E$ be an ordered field and let $F$ be a real closed subfield. We say that $E$ is $F$-archimedean if for each $ein E$ there is $xin F$ such that $-xle ele x$.




Is it true that if $E$ is $F$-archimedean then every interval in $E$ contains an element in $F$? That is, is it true that for every $e<e'$ in $E$ there is an element $xin F$ such that $e<x<e'$?






This is known if $F$ is the field of real algebraic numbers (in which case $E$ is an ordered subfield of $mathbb{R}$), and it seems to me that it should have an easy proof in the general case. However I cannot find neither an easy proof nor a counterexample.










share|cite|improve this question




















  • 2




    Have you tried $E = mathbb{Q}((infty)), F = mathbb{Q}((infty^2))$?
    – user44191
    Dec 17 at 19:49






  • 1




    What is $mathbb{Q}((infty))$? Do you mean $mathbb{Q}(t)$ with the order making $t$ an infinite positive element (I'm not sure this order can be extended to the Laurent series)?
    – Denis Nardin
    Dec 17 at 19:50










  • Yes, sorry, my idiocy.
    – user44191
    Dec 17 at 19:51










  • No need to be self-deprecating it's a good counterexample... Uhm there might in fact be no number between $t$ and $t+1$. Can you do a counterexample where $F$ is real closed? Sorry for changing the goal posts, I should have put it from the beginning since I was really thinking about that case.
    – Denis Nardin
    Dec 17 at 19:52








  • 1




    Try $E$ the real closure of $mathbb{Q}(x, y)$ with $x > mathbb{Q}, y > mathbb{Q}(x)$, and $F$ the real closure of $mathbb{Q}(y)$?
    – user44191
    Dec 17 at 20:00
















11












11








11







Let $E$ be an ordered field and let $F$ be a real closed subfield. We say that $E$ is $F$-archimedean if for each $ein E$ there is $xin F$ such that $-xle ele x$.




Is it true that if $E$ is $F$-archimedean then every interval in $E$ contains an element in $F$? That is, is it true that for every $e<e'$ in $E$ there is an element $xin F$ such that $e<x<e'$?






This is known if $F$ is the field of real algebraic numbers (in which case $E$ is an ordered subfield of $mathbb{R}$), and it seems to me that it should have an easy proof in the general case. However I cannot find neither an easy proof nor a counterexample.










share|cite|improve this question















Let $E$ be an ordered field and let $F$ be a real closed subfield. We say that $E$ is $F$-archimedean if for each $ein E$ there is $xin F$ such that $-xle ele x$.




Is it true that if $E$ is $F$-archimedean then every interval in $E$ contains an element in $F$? That is, is it true that for every $e<e'$ in $E$ there is an element $xin F$ such that $e<x<e'$?






This is known if $F$ is the field of real algebraic numbers (in which case $E$ is an ordered subfield of $mathbb{R}$), and it seems to me that it should have an easy proof in the general case. However I cannot find neither an easy proof nor a counterexample.







ac.commutative-algebra ordered-fields






share|cite|improve this question















share|cite|improve this question













share|cite|improve this question




share|cite|improve this question








edited Dec 17 at 19:53

























asked Dec 17 at 19:37









Denis Nardin

7,62213057




7,62213057








  • 2




    Have you tried $E = mathbb{Q}((infty)), F = mathbb{Q}((infty^2))$?
    – user44191
    Dec 17 at 19:49






  • 1




    What is $mathbb{Q}((infty))$? Do you mean $mathbb{Q}(t)$ with the order making $t$ an infinite positive element (I'm not sure this order can be extended to the Laurent series)?
    – Denis Nardin
    Dec 17 at 19:50










  • Yes, sorry, my idiocy.
    – user44191
    Dec 17 at 19:51










  • No need to be self-deprecating it's a good counterexample... Uhm there might in fact be no number between $t$ and $t+1$. Can you do a counterexample where $F$ is real closed? Sorry for changing the goal posts, I should have put it from the beginning since I was really thinking about that case.
    – Denis Nardin
    Dec 17 at 19:52








  • 1




    Try $E$ the real closure of $mathbb{Q}(x, y)$ with $x > mathbb{Q}, y > mathbb{Q}(x)$, and $F$ the real closure of $mathbb{Q}(y)$?
    – user44191
    Dec 17 at 20:00
















  • 2




    Have you tried $E = mathbb{Q}((infty)), F = mathbb{Q}((infty^2))$?
    – user44191
    Dec 17 at 19:49






  • 1




    What is $mathbb{Q}((infty))$? Do you mean $mathbb{Q}(t)$ with the order making $t$ an infinite positive element (I'm not sure this order can be extended to the Laurent series)?
    – Denis Nardin
    Dec 17 at 19:50










  • Yes, sorry, my idiocy.
    – user44191
    Dec 17 at 19:51










  • No need to be self-deprecating it's a good counterexample... Uhm there might in fact be no number between $t$ and $t+1$. Can you do a counterexample where $F$ is real closed? Sorry for changing the goal posts, I should have put it from the beginning since I was really thinking about that case.
    – Denis Nardin
    Dec 17 at 19:52








  • 1




    Try $E$ the real closure of $mathbb{Q}(x, y)$ with $x > mathbb{Q}, y > mathbb{Q}(x)$, and $F$ the real closure of $mathbb{Q}(y)$?
    – user44191
    Dec 17 at 20:00










2




2




Have you tried $E = mathbb{Q}((infty)), F = mathbb{Q}((infty^2))$?
– user44191
Dec 17 at 19:49




Have you tried $E = mathbb{Q}((infty)), F = mathbb{Q}((infty^2))$?
– user44191
Dec 17 at 19:49




1




1




What is $mathbb{Q}((infty))$? Do you mean $mathbb{Q}(t)$ with the order making $t$ an infinite positive element (I'm not sure this order can be extended to the Laurent series)?
– Denis Nardin
Dec 17 at 19:50




What is $mathbb{Q}((infty))$? Do you mean $mathbb{Q}(t)$ with the order making $t$ an infinite positive element (I'm not sure this order can be extended to the Laurent series)?
– Denis Nardin
Dec 17 at 19:50












Yes, sorry, my idiocy.
– user44191
Dec 17 at 19:51




Yes, sorry, my idiocy.
– user44191
Dec 17 at 19:51












No need to be self-deprecating it's a good counterexample... Uhm there might in fact be no number between $t$ and $t+1$. Can you do a counterexample where $F$ is real closed? Sorry for changing the goal posts, I should have put it from the beginning since I was really thinking about that case.
– Denis Nardin
Dec 17 at 19:52






No need to be self-deprecating it's a good counterexample... Uhm there might in fact be no number between $t$ and $t+1$. Can you do a counterexample where $F$ is real closed? Sorry for changing the goal posts, I should have put it from the beginning since I was really thinking about that case.
– Denis Nardin
Dec 17 at 19:52






1




1




Try $E$ the real closure of $mathbb{Q}(x, y)$ with $x > mathbb{Q}, y > mathbb{Q}(x)$, and $F$ the real closure of $mathbb{Q}(y)$?
– user44191
Dec 17 at 20:00






Try $E$ the real closure of $mathbb{Q}(x, y)$ with $x > mathbb{Q}, y > mathbb{Q}(x)$, and $F$ the real closure of $mathbb{Q}(y)$?
– user44191
Dec 17 at 20:00












3 Answers
3






active

oldest

votes


















10














Let $F$ be any real-closed field of uncountable cofinality. That is, every countable subset of $F$ is bounded. One can make such a field in a process of $omega_1$-many field extensions; alternatively, the ultrapower of $mathbb{R}$ by nonprincipal ultrafilter on $omega$ also has uncountable cofinality.



Let $E=F^omega/mu$ be an ultrapower of $F$ by a nonprincipal ultrafilter $mu$ on $omega$. We may identify $F$ with its canonical copy in $E$ using equivalence classes of constant functions $xmapsto [c_x]_mu$.



Since every function from $omega$ to $F$ is bounded by a constant function, it follows that $E$ is $F$-archimedean. But $F$ is not dense in $E$, since there are no constant functions between $[text{id}]_mu$ and $[text{id}+1]_mu$, where $text{id}:nmapsto n$, viewing $omegasubset F$.






share|cite|improve this answer



















  • 1




    Oh well... I guess that was too good to be true. Thanks for the answer, I'll wait till tomorrow to see if other interesting answers pop up and then accept it.
    – Denis Nardin
    Dec 17 at 20:08








  • 1




    Sure thing, no problem. I liked your question. The argument I give shows that no first-order property of the ordered field (like being real-closed) can have the consequence you want, since we can always find such fields of uncountable cofinality and then take an ultrapower.
    – Joel David Hamkins
    Dec 17 at 20:22



















13














Let $E$ be the real closure of $mathbb{Q}(x, y) = (mathbb{Q}(x))(y)$, with order given by $x > mathbb{Q}$ and$y > mathbb{Q}(x)$. In other words, positivity on $mathbb{Q}(x, y)$ is determined first by degree in $y$, and then by degree in $x$. Then let $F$ be the real closure of $mathbb{Q}(y)$.



First, we prove that $E$ is $F$-archimedean. Let $e in E$. There is some $e' in mathbb{Q}(x, y)$ with $e' > e > -e'$. Then $e'$ has degree $n$ in $y$ for some $y$; let $f = y^{n + 1}$. By the order on $mathbb{Q}(x, y)$, we have $f > e' > e > -e' > -f$. Therefore $E$ is $F$-archimedean.



On the other hand, clearly, there is no element of $F$ between $x$ and $x + 1$.






share|cite|improve this answer

















  • 1




    Thank you for your answer! Now I'm going to have a hard time deciding which one to accept :)
    – Denis Nardin
    Dec 17 at 20:35






  • 1




    I ended up accepting JDH's answer because it provides me with a "machine" to generate counterexamples to similar statements, but your simple answer was very appreciated (and possibly shows that I hadn't thought through the situation as much as I thought...)
    – Denis Nardin
    Dec 18 at 22:37



















5














This is similar to user44191's answer but I want to put the emphasis on the fact that there is no reason that the cofinality of $F$ in $E$ (which is what ou call [$E$ is $F$-archimedean]) imply the density of $F$ in $E$.



Indeed, if $F$ is any non-archimedean ordered field, then the field $F(t)$ can be equipped with an order where $t$ is positive infinite but smaller than any positive infinite element of $F$. Thus $F$ is cofinal in $F(t)$ but not dense in it since no element of $F$ is close to $t$. To do so, say that a fraction $frac{P(t)}{Q(t)}$ is positive if $P(t)$ and $Q(t)$ have the same sign, where the sign of a polynomial $R(t)$ is positive if $R(s)$ is positive for sufficiently large finite element $s$ of $F$.



This is a special case of filling a cut in an ordered field using a simple extension.






share|cite|improve this answer





















    Your Answer





    StackExchange.ifUsing("editor", function () {
    return StackExchange.using("mathjaxEditing", function () {
    StackExchange.MarkdownEditor.creationCallbacks.add(function (editor, postfix) {
    StackExchange.mathjaxEditing.prepareWmdForMathJax(editor, postfix, [["$", "$"], ["\\(","\\)"]]);
    });
    });
    }, "mathjax-editing");

    StackExchange.ready(function() {
    var channelOptions = {
    tags: "".split(" "),
    id: "504"
    };
    initTagRenderer("".split(" "), "".split(" "), channelOptions);

    StackExchange.using("externalEditor", function() {
    // Have to fire editor after snippets, if snippets enabled
    if (StackExchange.settings.snippets.snippetsEnabled) {
    StackExchange.using("snippets", function() {
    createEditor();
    });
    }
    else {
    createEditor();
    }
    });

    function createEditor() {
    StackExchange.prepareEditor({
    heartbeatType: 'answer',
    autoActivateHeartbeat: false,
    convertImagesToLinks: true,
    noModals: true,
    showLowRepImageUploadWarning: true,
    reputationToPostImages: 10,
    bindNavPrevention: true,
    postfix: "",
    imageUploader: {
    brandingHtml: "Powered by u003ca class="icon-imgur-white" href="https://imgur.com/"u003eu003c/au003e",
    contentPolicyHtml: "User contributions licensed under u003ca href="https://creativecommons.org/licenses/by-sa/3.0/"u003ecc by-sa 3.0 with attribution requiredu003c/au003e u003ca href="https://stackoverflow.com/legal/content-policy"u003e(content policy)u003c/au003e",
    allowUrls: true
    },
    noCode: true, onDemand: true,
    discardSelector: ".discard-answer"
    ,immediatelyShowMarkdownHelp:true
    });


    }
    });














    draft saved

    draft discarded


















    StackExchange.ready(
    function () {
    StackExchange.openid.initPostLogin('.new-post-login', 'https%3a%2f%2fmathoverflow.net%2fquestions%2f318901%2fare-archimedean-subextensions-of-ordered-fields-dense%23new-answer', 'question_page');
    }
    );

    Post as a guest















    Required, but never shown

























    3 Answers
    3






    active

    oldest

    votes








    3 Answers
    3






    active

    oldest

    votes









    active

    oldest

    votes






    active

    oldest

    votes









    10














    Let $F$ be any real-closed field of uncountable cofinality. That is, every countable subset of $F$ is bounded. One can make such a field in a process of $omega_1$-many field extensions; alternatively, the ultrapower of $mathbb{R}$ by nonprincipal ultrafilter on $omega$ also has uncountable cofinality.



    Let $E=F^omega/mu$ be an ultrapower of $F$ by a nonprincipal ultrafilter $mu$ on $omega$. We may identify $F$ with its canonical copy in $E$ using equivalence classes of constant functions $xmapsto [c_x]_mu$.



    Since every function from $omega$ to $F$ is bounded by a constant function, it follows that $E$ is $F$-archimedean. But $F$ is not dense in $E$, since there are no constant functions between $[text{id}]_mu$ and $[text{id}+1]_mu$, where $text{id}:nmapsto n$, viewing $omegasubset F$.






    share|cite|improve this answer



















    • 1




      Oh well... I guess that was too good to be true. Thanks for the answer, I'll wait till tomorrow to see if other interesting answers pop up and then accept it.
      – Denis Nardin
      Dec 17 at 20:08








    • 1




      Sure thing, no problem. I liked your question. The argument I give shows that no first-order property of the ordered field (like being real-closed) can have the consequence you want, since we can always find such fields of uncountable cofinality and then take an ultrapower.
      – Joel David Hamkins
      Dec 17 at 20:22
















    10














    Let $F$ be any real-closed field of uncountable cofinality. That is, every countable subset of $F$ is bounded. One can make such a field in a process of $omega_1$-many field extensions; alternatively, the ultrapower of $mathbb{R}$ by nonprincipal ultrafilter on $omega$ also has uncountable cofinality.



    Let $E=F^omega/mu$ be an ultrapower of $F$ by a nonprincipal ultrafilter $mu$ on $omega$. We may identify $F$ with its canonical copy in $E$ using equivalence classes of constant functions $xmapsto [c_x]_mu$.



    Since every function from $omega$ to $F$ is bounded by a constant function, it follows that $E$ is $F$-archimedean. But $F$ is not dense in $E$, since there are no constant functions between $[text{id}]_mu$ and $[text{id}+1]_mu$, where $text{id}:nmapsto n$, viewing $omegasubset F$.






    share|cite|improve this answer



















    • 1




      Oh well... I guess that was too good to be true. Thanks for the answer, I'll wait till tomorrow to see if other interesting answers pop up and then accept it.
      – Denis Nardin
      Dec 17 at 20:08








    • 1




      Sure thing, no problem. I liked your question. The argument I give shows that no first-order property of the ordered field (like being real-closed) can have the consequence you want, since we can always find such fields of uncountable cofinality and then take an ultrapower.
      – Joel David Hamkins
      Dec 17 at 20:22














    10












    10








    10






    Let $F$ be any real-closed field of uncountable cofinality. That is, every countable subset of $F$ is bounded. One can make such a field in a process of $omega_1$-many field extensions; alternatively, the ultrapower of $mathbb{R}$ by nonprincipal ultrafilter on $omega$ also has uncountable cofinality.



    Let $E=F^omega/mu$ be an ultrapower of $F$ by a nonprincipal ultrafilter $mu$ on $omega$. We may identify $F$ with its canonical copy in $E$ using equivalence classes of constant functions $xmapsto [c_x]_mu$.



    Since every function from $omega$ to $F$ is bounded by a constant function, it follows that $E$ is $F$-archimedean. But $F$ is not dense in $E$, since there are no constant functions between $[text{id}]_mu$ and $[text{id}+1]_mu$, where $text{id}:nmapsto n$, viewing $omegasubset F$.






    share|cite|improve this answer














    Let $F$ be any real-closed field of uncountable cofinality. That is, every countable subset of $F$ is bounded. One can make such a field in a process of $omega_1$-many field extensions; alternatively, the ultrapower of $mathbb{R}$ by nonprincipal ultrafilter on $omega$ also has uncountable cofinality.



    Let $E=F^omega/mu$ be an ultrapower of $F$ by a nonprincipal ultrafilter $mu$ on $omega$. We may identify $F$ with its canonical copy in $E$ using equivalence classes of constant functions $xmapsto [c_x]_mu$.



    Since every function from $omega$ to $F$ is bounded by a constant function, it follows that $E$ is $F$-archimedean. But $F$ is not dense in $E$, since there are no constant functions between $[text{id}]_mu$ and $[text{id}+1]_mu$, where $text{id}:nmapsto n$, viewing $omegasubset F$.







    share|cite|improve this answer














    share|cite|improve this answer



    share|cite|improve this answer








    edited Dec 17 at 20:09

























    answered Dec 17 at 20:04









    Joel David Hamkins

    163k25501863




    163k25501863








    • 1




      Oh well... I guess that was too good to be true. Thanks for the answer, I'll wait till tomorrow to see if other interesting answers pop up and then accept it.
      – Denis Nardin
      Dec 17 at 20:08








    • 1




      Sure thing, no problem. I liked your question. The argument I give shows that no first-order property of the ordered field (like being real-closed) can have the consequence you want, since we can always find such fields of uncountable cofinality and then take an ultrapower.
      – Joel David Hamkins
      Dec 17 at 20:22














    • 1




      Oh well... I guess that was too good to be true. Thanks for the answer, I'll wait till tomorrow to see if other interesting answers pop up and then accept it.
      – Denis Nardin
      Dec 17 at 20:08








    • 1




      Sure thing, no problem. I liked your question. The argument I give shows that no first-order property of the ordered field (like being real-closed) can have the consequence you want, since we can always find such fields of uncountable cofinality and then take an ultrapower.
      – Joel David Hamkins
      Dec 17 at 20:22








    1




    1




    Oh well... I guess that was too good to be true. Thanks for the answer, I'll wait till tomorrow to see if other interesting answers pop up and then accept it.
    – Denis Nardin
    Dec 17 at 20:08






    Oh well... I guess that was too good to be true. Thanks for the answer, I'll wait till tomorrow to see if other interesting answers pop up and then accept it.
    – Denis Nardin
    Dec 17 at 20:08






    1




    1




    Sure thing, no problem. I liked your question. The argument I give shows that no first-order property of the ordered field (like being real-closed) can have the consequence you want, since we can always find such fields of uncountable cofinality and then take an ultrapower.
    – Joel David Hamkins
    Dec 17 at 20:22




    Sure thing, no problem. I liked your question. The argument I give shows that no first-order property of the ordered field (like being real-closed) can have the consequence you want, since we can always find such fields of uncountable cofinality and then take an ultrapower.
    – Joel David Hamkins
    Dec 17 at 20:22











    13














    Let $E$ be the real closure of $mathbb{Q}(x, y) = (mathbb{Q}(x))(y)$, with order given by $x > mathbb{Q}$ and$y > mathbb{Q}(x)$. In other words, positivity on $mathbb{Q}(x, y)$ is determined first by degree in $y$, and then by degree in $x$. Then let $F$ be the real closure of $mathbb{Q}(y)$.



    First, we prove that $E$ is $F$-archimedean. Let $e in E$. There is some $e' in mathbb{Q}(x, y)$ with $e' > e > -e'$. Then $e'$ has degree $n$ in $y$ for some $y$; let $f = y^{n + 1}$. By the order on $mathbb{Q}(x, y)$, we have $f > e' > e > -e' > -f$. Therefore $E$ is $F$-archimedean.



    On the other hand, clearly, there is no element of $F$ between $x$ and $x + 1$.






    share|cite|improve this answer

















    • 1




      Thank you for your answer! Now I'm going to have a hard time deciding which one to accept :)
      – Denis Nardin
      Dec 17 at 20:35






    • 1




      I ended up accepting JDH's answer because it provides me with a "machine" to generate counterexamples to similar statements, but your simple answer was very appreciated (and possibly shows that I hadn't thought through the situation as much as I thought...)
      – Denis Nardin
      Dec 18 at 22:37
















    13














    Let $E$ be the real closure of $mathbb{Q}(x, y) = (mathbb{Q}(x))(y)$, with order given by $x > mathbb{Q}$ and$y > mathbb{Q}(x)$. In other words, positivity on $mathbb{Q}(x, y)$ is determined first by degree in $y$, and then by degree in $x$. Then let $F$ be the real closure of $mathbb{Q}(y)$.



    First, we prove that $E$ is $F$-archimedean. Let $e in E$. There is some $e' in mathbb{Q}(x, y)$ with $e' > e > -e'$. Then $e'$ has degree $n$ in $y$ for some $y$; let $f = y^{n + 1}$. By the order on $mathbb{Q}(x, y)$, we have $f > e' > e > -e' > -f$. Therefore $E$ is $F$-archimedean.



    On the other hand, clearly, there is no element of $F$ between $x$ and $x + 1$.






    share|cite|improve this answer

















    • 1




      Thank you for your answer! Now I'm going to have a hard time deciding which one to accept :)
      – Denis Nardin
      Dec 17 at 20:35






    • 1




      I ended up accepting JDH's answer because it provides me with a "machine" to generate counterexamples to similar statements, but your simple answer was very appreciated (and possibly shows that I hadn't thought through the situation as much as I thought...)
      – Denis Nardin
      Dec 18 at 22:37














    13












    13








    13






    Let $E$ be the real closure of $mathbb{Q}(x, y) = (mathbb{Q}(x))(y)$, with order given by $x > mathbb{Q}$ and$y > mathbb{Q}(x)$. In other words, positivity on $mathbb{Q}(x, y)$ is determined first by degree in $y$, and then by degree in $x$. Then let $F$ be the real closure of $mathbb{Q}(y)$.



    First, we prove that $E$ is $F$-archimedean. Let $e in E$. There is some $e' in mathbb{Q}(x, y)$ with $e' > e > -e'$. Then $e'$ has degree $n$ in $y$ for some $y$; let $f = y^{n + 1}$. By the order on $mathbb{Q}(x, y)$, we have $f > e' > e > -e' > -f$. Therefore $E$ is $F$-archimedean.



    On the other hand, clearly, there is no element of $F$ between $x$ and $x + 1$.






    share|cite|improve this answer












    Let $E$ be the real closure of $mathbb{Q}(x, y) = (mathbb{Q}(x))(y)$, with order given by $x > mathbb{Q}$ and$y > mathbb{Q}(x)$. In other words, positivity on $mathbb{Q}(x, y)$ is determined first by degree in $y$, and then by degree in $x$. Then let $F$ be the real closure of $mathbb{Q}(y)$.



    First, we prove that $E$ is $F$-archimedean. Let $e in E$. There is some $e' in mathbb{Q}(x, y)$ with $e' > e > -e'$. Then $e'$ has degree $n$ in $y$ for some $y$; let $f = y^{n + 1}$. By the order on $mathbb{Q}(x, y)$, we have $f > e' > e > -e' > -f$. Therefore $E$ is $F$-archimedean.



    On the other hand, clearly, there is no element of $F$ between $x$ and $x + 1$.







    share|cite|improve this answer












    share|cite|improve this answer



    share|cite|improve this answer










    answered Dec 17 at 20:22









    user44191

    2,5431126




    2,5431126








    • 1




      Thank you for your answer! Now I'm going to have a hard time deciding which one to accept :)
      – Denis Nardin
      Dec 17 at 20:35






    • 1




      I ended up accepting JDH's answer because it provides me with a "machine" to generate counterexamples to similar statements, but your simple answer was very appreciated (and possibly shows that I hadn't thought through the situation as much as I thought...)
      – Denis Nardin
      Dec 18 at 22:37














    • 1




      Thank you for your answer! Now I'm going to have a hard time deciding which one to accept :)
      – Denis Nardin
      Dec 17 at 20:35






    • 1




      I ended up accepting JDH's answer because it provides me with a "machine" to generate counterexamples to similar statements, but your simple answer was very appreciated (and possibly shows that I hadn't thought through the situation as much as I thought...)
      – Denis Nardin
      Dec 18 at 22:37








    1




    1




    Thank you for your answer! Now I'm going to have a hard time deciding which one to accept :)
    – Denis Nardin
    Dec 17 at 20:35




    Thank you for your answer! Now I'm going to have a hard time deciding which one to accept :)
    – Denis Nardin
    Dec 17 at 20:35




    1




    1




    I ended up accepting JDH's answer because it provides me with a "machine" to generate counterexamples to similar statements, but your simple answer was very appreciated (and possibly shows that I hadn't thought through the situation as much as I thought...)
    – Denis Nardin
    Dec 18 at 22:37




    I ended up accepting JDH's answer because it provides me with a "machine" to generate counterexamples to similar statements, but your simple answer was very appreciated (and possibly shows that I hadn't thought through the situation as much as I thought...)
    – Denis Nardin
    Dec 18 at 22:37











    5














    This is similar to user44191's answer but I want to put the emphasis on the fact that there is no reason that the cofinality of $F$ in $E$ (which is what ou call [$E$ is $F$-archimedean]) imply the density of $F$ in $E$.



    Indeed, if $F$ is any non-archimedean ordered field, then the field $F(t)$ can be equipped with an order where $t$ is positive infinite but smaller than any positive infinite element of $F$. Thus $F$ is cofinal in $F(t)$ but not dense in it since no element of $F$ is close to $t$. To do so, say that a fraction $frac{P(t)}{Q(t)}$ is positive if $P(t)$ and $Q(t)$ have the same sign, where the sign of a polynomial $R(t)$ is positive if $R(s)$ is positive for sufficiently large finite element $s$ of $F$.



    This is a special case of filling a cut in an ordered field using a simple extension.






    share|cite|improve this answer


























      5














      This is similar to user44191's answer but I want to put the emphasis on the fact that there is no reason that the cofinality of $F$ in $E$ (which is what ou call [$E$ is $F$-archimedean]) imply the density of $F$ in $E$.



      Indeed, if $F$ is any non-archimedean ordered field, then the field $F(t)$ can be equipped with an order where $t$ is positive infinite but smaller than any positive infinite element of $F$. Thus $F$ is cofinal in $F(t)$ but not dense in it since no element of $F$ is close to $t$. To do so, say that a fraction $frac{P(t)}{Q(t)}$ is positive if $P(t)$ and $Q(t)$ have the same sign, where the sign of a polynomial $R(t)$ is positive if $R(s)$ is positive for sufficiently large finite element $s$ of $F$.



      This is a special case of filling a cut in an ordered field using a simple extension.






      share|cite|improve this answer
























        5












        5








        5






        This is similar to user44191's answer but I want to put the emphasis on the fact that there is no reason that the cofinality of $F$ in $E$ (which is what ou call [$E$ is $F$-archimedean]) imply the density of $F$ in $E$.



        Indeed, if $F$ is any non-archimedean ordered field, then the field $F(t)$ can be equipped with an order where $t$ is positive infinite but smaller than any positive infinite element of $F$. Thus $F$ is cofinal in $F(t)$ but not dense in it since no element of $F$ is close to $t$. To do so, say that a fraction $frac{P(t)}{Q(t)}$ is positive if $P(t)$ and $Q(t)$ have the same sign, where the sign of a polynomial $R(t)$ is positive if $R(s)$ is positive for sufficiently large finite element $s$ of $F$.



        This is a special case of filling a cut in an ordered field using a simple extension.






        share|cite|improve this answer












        This is similar to user44191's answer but I want to put the emphasis on the fact that there is no reason that the cofinality of $F$ in $E$ (which is what ou call [$E$ is $F$-archimedean]) imply the density of $F$ in $E$.



        Indeed, if $F$ is any non-archimedean ordered field, then the field $F(t)$ can be equipped with an order where $t$ is positive infinite but smaller than any positive infinite element of $F$. Thus $F$ is cofinal in $F(t)$ but not dense in it since no element of $F$ is close to $t$. To do so, say that a fraction $frac{P(t)}{Q(t)}$ is positive if $P(t)$ and $Q(t)$ have the same sign, where the sign of a polynomial $R(t)$ is positive if $R(s)$ is positive for sufficiently large finite element $s$ of $F$.



        This is a special case of filling a cut in an ordered field using a simple extension.







        share|cite|improve this answer












        share|cite|improve this answer



        share|cite|improve this answer










        answered 2 days ago









        nombre

        7281612




        7281612






























            draft saved

            draft discarded




















































            Thanks for contributing an answer to MathOverflow!


            • Please be sure to answer the question. Provide details and share your research!

            But avoid



            • Asking for help, clarification, or responding to other answers.

            • Making statements based on opinion; back them up with references or personal experience.


            Use MathJax to format equations. MathJax reference.


            To learn more, see our tips on writing great answers.





            Some of your past answers have not been well-received, and you're in danger of being blocked from answering.


            Please pay close attention to the following guidance:


            • Please be sure to answer the question. Provide details and share your research!

            But avoid



            • Asking for help, clarification, or responding to other answers.

            • Making statements based on opinion; back them up with references or personal experience.


            To learn more, see our tips on writing great answers.




            draft saved


            draft discarded














            StackExchange.ready(
            function () {
            StackExchange.openid.initPostLogin('.new-post-login', 'https%3a%2f%2fmathoverflow.net%2fquestions%2f318901%2fare-archimedean-subextensions-of-ordered-fields-dense%23new-answer', 'question_page');
            }
            );

            Post as a guest















            Required, but never shown





















































            Required, but never shown














            Required, but never shown












            Required, but never shown







            Required, but never shown

































            Required, but never shown














            Required, but never shown












            Required, but never shown







            Required, but never shown







            Popular posts from this blog

            If I really need a card on my start hand, how many mulligans make sense? [duplicate]

            Alcedinidae

            Can an atomic nucleus contain both particles and antiparticles? [duplicate]